Difference between revisions of "1988 AJHSME Problems/Problem 7"

(Solution)
(Solution)
Line 9: Line 9:
 
We estimate the first thing to be <math>2.5</math>, the second thing to be <math>8</math>, and the third thing to be <math>10</math>. We now have <math>2.5\cdot 8\cdot 10=25\cdot 8=200\Rightarrow \mathrm{(B)}</math>.
 
We estimate the first thing to be <math>2.5</math>, the second thing to be <math>8</math>, and the third thing to be <math>10</math>. We now have <math>2.5\cdot 8\cdot 10=25\cdot 8=200\Rightarrow \mathrm{(B)}</math>.
  
Note: The real answer is 200.789719 ~ xxsc
+
Note: The real answer is 200.78971902 ~ xxsc
  
 
==See Also==
 
==See Also==

Revision as of 14:01, 12 November 2019

Problem

$2.46\times 8.163\times (5.17+4.829)$ is closest to

$\text{(A)}\ 100 \qquad \text{(B)}\ 200 \qquad \text{(C)}\ 300 \qquad \text{(D)}\ 400 \qquad \text{(E)}\ 500$

Solution

We estimate the first thing to be $2.5$, the second thing to be $8$, and the third thing to be $10$. We now have $2.5\cdot 8\cdot 10=25\cdot 8=200\Rightarrow \mathrm{(B)}$.

Note: The real answer is 200.78971902 ~ xxsc

See Also

1988 AJHSME (ProblemsAnswer KeyResources)
Preceded by
Problem 6
Followed by
Problem 8
1 2 3 4 5 6 7 8 9 10 11 12 13 14 15 16 17 18 19 20 21 22 23 24 25
All AJHSME/AMC 8 Problems and Solutions

The problems on this page are copyrighted by the Mathematical Association of America's American Mathematics Competitions. AMC logo.png